What is the effect on the graph of the function f(x) = x2 when f(x) is changed to f( 1 3 x) + 6? A) stretched horizontally and shifted up B) stretched vertically and shifted right C) compressed vertically and shifted left Eliminate D) compressed horizontally and shifted down

Answers

Answer 1

Answer:

A

Step-by-step explanation:

f(x) = x²

f(x) becomes f(13x) + 6

the "+ 6" at the end tells us already everything, because there is only one aber option listed that contains a shift up. and that is exactly what that "+ 6" did. it adds 6 to every possible y value of the function. so, the whole function moves up by 6 units.

and yes, the 13x of f(13x) stretches the function horizontally (along the x axis).

simply said, whenever the x value for the original function increased by 1, it increases for the new function by 13.


Related Questions

can someone please help mee and graph it for me pleasee(20 points and i will give brainliest!!!)

Answers

Answer:

vertex: (3,1)

y-intercept = (0,-8)

x-intercept- Smaller=2 Larger=4

Domain= all real numbers, identity, infinite

Range= 1 is greater than or equal to y

axis of symmetry=3

Step-by-step explanation:

In the diagram shown, the area of the triangle is 44.3m². If b = 14.1m and the angle at C = 48°, work out x. Round to 1 DP. X C b​

Answers

If the height of the triangle will be 6.28 meters. Then the value of x will be 8.46 meters.

What is the triangle?

A triangle is a three-sided polygon with three angles. The angles of the triangle add up to 180 degrees.

In the diagram shown, the area of the triangle is 44.3 m².

If b = 14.1 m and the angle at C = 48°.

Then the value of x will be

The height of the triangle will be

    A = 1/2 × B × H

44.3 = 1/2 × 14.1 × h

    h = 6.28 m

Then the value of x will be

sin 48° = 6.28 / x

x = 8.455 ≈ 8.46 m

More about the triangle link is given below.

https://brainly.com/question/25813512

#SPJ1

can someone please help mee

Answers

[tex]\quad \huge \quad \quad \boxed{ \tt \:Answer }[/tex]

[tex]\qquad \tt \rightarrow \:Domain = [-2, 1)[/tex]

[tex]\qquad \tt \rightarrow \:Range = [0 , 4][/tex]

____________________________________

[tex] \large \tt Solution \: : [/tex]

Domain = All possible values of x for which f(x) is defined

[ generally the extension of function in x - direction ]

Range = All possible values of f(x)

[ generally the extension of function in y - direction ]

[tex] \large\textsf{For the given graph : } [/tex]

[tex]\qquad \tt \rightarrow \: domain = [ - 2,1)[/tex]

[tex]\qquad \tt \rightarrow \: range= [ 0,4][/tex]

Answered by : ❝ AǫᴜᴀWɪᴢ ❞

The number of boys who signed up to play winter basketball is 175 percent of what it was last year. if 320 boys played winter basketball last year, how many signed up to play this year?

Answers

The number of boys signed up to play basketball this year is 560.

Given that, the number of boys who played basketball last year = 320.

We need to find the number of boys signed up to play basketball this year.

What is a percentage?

In mathematics, a percentage is a number or ratio expressed as a fraction of 100.

Now, the number of boys signed up to play basketball this year = 175 per cent of what it was last year.

So, the number of boys signed up to play basketball this year = 175% of 320

[tex]=\frac{175}{10} \times 320=1.75 \times 320=560[/tex]

Therefore, the number of boys signed up to play basketball this year is 560.

To learn more about percentages visit:

https://brainly.com/question/13450942.

#SPJ1

23.
(05.07 MC)

Use the data in the table to determine the correlation coefficient. (1 point)

Overall Student Average Math Class Average
90 80
95 90
80 90
84 95
75 75
80 85


0.65

0.35

0.30

0.03

Answers

The correlation coefficient of the data given in the table, using a calculator, is of 0.35

How to find the correlation coefficient of a data-set using a calculator?

To find the coefficient, we need to insert the points (x,y) in the calculator.

In this problem, we have that:

The values of x are: 90, 95, 80, 84, 75, 80.The values of y are: 80, 90, 90, 95, 75, 85.

Using a calculator, the coefficient is of 0.35.

More can be learned about correlation coefficients at https://brainly.com/question/25815006

#SPJ1

Please help solve for x..

Answers

Answer:

x = 1 or x = -1

Step-by-step explanation:

Given equation:

[tex]x^{100}-4^x \cdot x^{98}-x^2+4^x=0[/tex]

Factor out -1:

[tex]\implies -1(-x^{100}+4^x \cdot x^{98}+x^2-4^x)=0[/tex]

Divide both sides by -1:

[tex]\implies -x^{100}+4^x \cdot x^{98}+x^2-4^x=0[/tex]

Rearrange the terms:

[tex]\implies 4^x \cdot x^{98}-4^x-x^{100}+x^2=0[/tex]

[tex]\textsf{Apply exponent rule} \quad a^{b+c}=a^b \cdot a^c \quad \textsf{to }x^{100}:[/tex]

[tex]\implies 4^x \cdot x^{98}-4^x-x^{98}x^2+x^2=0[/tex]

Factor the first two terms and the last two terms separately:

[tex]\implies 4^x(x^{98}-1)-x^2(x^{98}-1)=0[/tex]

Factor out the common term [tex](x^{98}-1)[/tex] :

[tex]\implies (4^x-x^2)(x^{98}-1)=0[/tex]

Zero Product Property:  If a ⋅ b = 0 then either a = 0 or b = 0 (or both).

Using the Zero Product Property, set each factor equal to zero and solve for x (if possible):

[tex]\begin{aligned}x^{98}-1 & = 0 & \quad \textsf{or} \quad \quad4^x-x^2 & = 0 \\x^{98} & =1 & 4^x & = x^2 \\x & = 1, -1 & \textsf{no}& \textsf{ solutions for } x \in \mathbb{R}\end{aligned}[/tex]

Therefore, the solutions to the given equation are: x = 1 or x = -1

Learn more here:

https://brainly.com/question/27751281

https://brainly.com/question/21186424

Estimate the Value of 196/0.499

Answers

Answer:

The value of the expression will be "392.79".

Step-by-step explanation:

~ The technique of dividing a quantities into equivalent portions as well as    determining however many equal components may be formed, is determined as a division.

~ It is denoted by "/".

According to the equation, the expression is:

→ 196/0.499

By applying the division, we get

→ 392.79

Answer:

392

Explanation:

Note that we are estimating, not finding the actual value. 0.499 is very close to 0.5, so to estimate this expression, we can do 196 ÷ 0.5. Dividing a number by 0.5 is the same as doubling it, so this can also be seen as
196 · 2, which is 392.

Hope this helps !! :D
The actual answer is the other one, but again, you're finding an estimate!

These points are linear.
Find the slope.
x
012345
y 3 6 9 12 15 18
slope = [?]

Answers

Answer:

Step-by-step explanation:

If 5 hours at a computer terminal costs $25, what is the cost of 6 hours?

A. $30
B. $45
C. $150
D. $12

Answers

Answer:

30 dollars

Step-by-step explanation:

25/5 = 5 dollars per hour

so 6 hours would be 5 * 6 = 30 dollars

Answer:

It would be B

Step-by-step explanation:

First, we need to find out the unit rate

5/25=1/5

Therefore, that means in one hour the cost would be $5

Since you are going from one hour to 6 hours you need to multiply both numbers by 6

Since 5x6=30, it would be 30$ :)

Bearing a to b is 280 what is bearing b to A

Answers

Answer:

please see photo attached for detailed analysis.

a sunflower has increased in height from 72.4cm to 86.2 cm. what is the percentage increase?

Answers

Answer:

initial hieght = 72.4 cm

final hieght = 86.2 cm

change in hieght = 9.8 cm

percentage increase = (9.8 ÷ 72.4 ) * 100 = 13.59 %

The data set represents the number of minutes dan spent on his homework each night. 30, 35, 35, 45, 46, 46, 52, 55, 57 which box plot correctly represents the data?.

Answers

The box plot first represents the data set option first is correct the third quartile lies between 52 to 54.

What is the box and whisker plot?

A box and whisker plot is a method of abstracting a set of data that is approximated using an interval scale. It's also known as a box plot. These are primarily used to interpret data.

The question is incomplete.

The complete question is in the picture, please refer to the attached picture.

We have data set:

30, 35, 35, 45, 46, 46, 52, 55, 57

We can find below details from the data set:

Population size = 9

Minimum: 30

First quartile: 35

Median = 46

Third quartile = 53.5

Interquartile Range = 18.5

Maximum = 57

Outliers = none

Thus, the box plot first represents the data set option first is correct.

Learn more about the box and whisker plot here:

brainly.com/question/3209282

#SPJ1

Solve for x
((x + 3)/4) + (((2x - 12) - 1)/3) = 1

Answers

Answer:

x=5

Step-by-step explanation:

[tex]((x+3)/4)+(((2x-12)-1)/3)=1\\\frac{x+3}{4} +\frac{2x-12-1}{3} =1\\\frac{x+3}{4} +\frac{2x-13}{3} =1\\[/tex]

Now we have to cross multiply the denominator to progress further.

[tex]\frac{3(x+3)}{4*3} +\frac{4(2x-13)}{3*4} =1\\\frac{3x+9}{12} +\frac{8x-52}{12} =1\\\frac{3x+9+(8x-52)}{12} =1\\\frac{3x+9+8x-52}{12} =1\\\frac{11x-43}{12} =1\\11x-43=12\\11x=43+12\\11x=55\\x=\frac{55}{11} \\=5[/tex]

Answer: 5 x 4/7

Full explanation.

Answers

ANSWER:
5/1 x 4/7 (Cross Multiply) =20/7

Answer:

20/7

Step-by-step explanation:

5×4/7

numerator5×numerator4=20

denominator 7×denominator1=hence,answer is

20/7

Which of the following describes point D?
O (-4,0)
O (4,0)
O (0,4)
O (0,-4)

Answers

Answer:

The answer is C (0,4)

Step-by-step explanation:

Looking at the graph we have to find a point (x,y) of D.

Find point D. It is on the y-axis, but which one? Positive/Negative y-axis.

It appears to be on the positive y-axis.

Now we have to choose either option C or D.

Option C seems like the best choice since the point is (x,y) is (0,4) on the positive y-axis.

Option D is quite the opposite of point D since the point (0, -4) is on the negative y-axis.

The answer is C. (0,4)

Jacob traveled 171 miles in 3.8 hours. He wants to know how many miles he traveled in one hour, so he set up this problem:

Answers

Answer:

so first we have to divide each side by 38

4.5miles=0.1

45miles=1 hour

Hope This Helps!!!

Which of the following did you include in your response? Check all of the boxes that apply. the range, or spread, of the salaries whether most people make a very low salary and one or two people make a high salary the minimum salary, and whether that is an entry level or starting salary the maximum salary whether the maximum salary is a lot higher than the average and other salaries are a lot lower than the mean

Answers

Answer:

Check all of them

Step-by-step explanation:

If you are on Edg, just check all of them.

2x+y=8
y+3z=5
z+2w=1
5w+3x=9

what is x,y,z,w

Answers

Answer:

Systems of equations:

x=3

y=2

z=1

w=0

please narrate every step you make and make a 2 column proof!

thank you!!

Answers

Hang on ,it is already proven through given .

So The proof is of one line only

ray FEH bisects <DFG (Given)

Hence proved .

I need hard and easy math questions with the answer, i need 10 questions

thnx

Answers

Answer:

https://www.khanacademy.org/

Step-by-step explanation:

I am not entirely sure what you mean however if you sign up for this website and go to whichever grade level you need you can practice all you want

(c) The result of a number, when increased by 40%, is 2.1. Find the number. ( c ) The result of a number , when increased by 40 % , is 2.1 . Find the number .​

Answers

Answer:

1.5 is the number.

Explanation:

Let the number be 'n', this is 100% - original number.

When increased by 40%, 100% + 40% = 140%.

140% × n = 2.1

1.4n = 2.1

n = 2.1/1.4

n = 1.5

Hence, the original number is 1.5.

Please answer!!

-3 + 4/7 - 1/5

Answers

Step-by-step explanation:

please mark me as brainlest

Answer:

-92/35

Step-by-step explanation:

-3 + 4/7 - 1/5 =

-21/7 + 4/7 - 1/5 =

-(21*5) / (7*5) + (4*5)/(7*5) - (1*7)/(5*7) =

-105/35 + 20/35 - 7/35 =

-105+20-7 / 35 = -92/35

What is the measure of the unknown angle?

straight angle divided into a one hundred thirty two degree angle and an unknown angle

48°
49°
51°
53°

Answers

The answer would be 48 because a straight line has an angle of 180 so
180-132=48

one angle of a triangle measures 80° more than the smallest, while a third angle is twice the smallest. whats the measure of each angle?

Answers

Answer: The measure of each angle is  105°, 50°, and 25°.

Step-by-step explanation: The first step in solving this is writing down what you know.

In the phrase it tells you that "one angle of a triangle measures 80 more than the smallest", so we can say that one angle = 80 + smallest angle.

This can be simplified more to x = 80 + y

x = One Angle

y = Smallest Angle

Then if we continue with the phrase it says "a third angle is twice the smallest", so we can say that a third angle = 2(Smallest Angle).

This can be simplified more to z = 2(y)

z = Third Angle

We also know that a triangles angles add up to 180°, so we know x + y + z = 180.

So, with these equations we can now solve for x, y, and z.

The easiest way would be to simply just plug in x and z straight into the 180 equation as follows.

x = 80 + y

z = 2(y)

180 = x + y + z

180 = (80 + y) + y + (2y)

180 = 80 + 4y

100 = 4y

y = 25

Then we can use y to solve for the other two equations.

z = 2(25)     x = 80 + 25

z = 50         x = 105

After completing that you are left with the three angles of the triangle, 25, 50, and 105.

Understand equivalent ratios in the real world
Ani thinks the perfect milkshake has 5 scoops of ice cream for every 3 spoonfuls of caramel. Freeze Zone makes
"batches of milkshakes with 10 scoops of ice cream and 7 spoonfuls of caramel.
What will Ani think about the amount of caramel in Freeze Zone's milkshakes?
Choose 1 answer:
Freeze Zone's milkshakes have too much caramel.
Freeze Zone's milkshakes have too little caramel.
Freeze Zone's milkshakes are just right.

Answers

Answer:

They have too much caramel

Step-by-step explanation:

The ratio to the perfect milkshake would be 5:3 (5 ice-cream, 3 caramel)

The ratio for freeze zones milkshakes are 10:7 (10 ice-cream, 7 caramel)

As you can see the amount of has ice-cream has doubled (5x2=10)

but the amount of caramel has increased more than double (double would be 3x2=6 but its not it is 7)

Therfore there is too much caramel for the 'perfect milkshake'

Hope this helps :)

Answer:

It is she will think there is too much caramel

Step-by-step explanation:

This is because the ratio of 5:10 is 1:2, so you will have to multiply 3 by 2, which is 6, which is smaller than 7.

Use the formula to solve the equation x^2 + 9x + 8 = 0.

Answers

Answer:

[tex]x=-8, -1[/tex]

Step-by-step explanation:

Given the equation [tex]x^{2} +9x+8=0\\[/tex],

By the quadratic formula,

[tex]x=\frac{-b+-\sqrt{b^{2}-4ac } }{2a}[/tex],

[tex]x=\frac{-(9)+-\sqrt{(9)^{2}-4(1)(8) } }{2(1)}=\frac{-9+-\sqrt{49 } }{2}=\frac{-9+-7 }{2}[/tex]

We have two different solutions for x where [tex]x=\frac{-9-7 }{2}[/tex] and [tex]x=\frac{-9+7 }{2}[/tex]

So, x = [tex]-16/2=-8[/tex] and [tex]x=-2/2=-1[/tex]

Therefore, the solutions to this quadratic equation is x = -8, -1, by the quadratic formula.

The equation of a line is y = 1.5x − 2. What are its slope and y-intercept?

Answers

You need to use y=mx+c to answer this so basically

gradient/slope (m) is 1.5 and the y-intercept (c) is -2

Hope this helps :)

[tex]\triangleright[/tex]Hii! [tex]\triangleleft[/tex]

-- - -- - -- - -- - -- - - - -- - -- - -- - -- - -- - -- - -- -- -- - -- -- - -- -- -- -- --- -- -- --- - -- -- --

[tex]\triangleright[/tex] [tex]\stackrel\circ{\rightsquigarrow\circ\boldsymbol{\underbrace{Answer}}}\circ\leftharpoonup[/tex] [tex]\triangleleft[/tex]

Slope = 1.5 ✅Y intercept = -2 ✅

[tex]\stackrel\circ{\rightsquigarrow\circ\boldsymbol{\underbrace{Explanation}}}\circ\leftharpoonup[/tex]

[tex]\bullet[/tex] Let's start by examining the equation we are working with more closely

-- ↩

This particular equation is provided to us in "slope intercept form".

--

The slope intercept formula is "[tex]\boldsymbol{y=mx+c}[/tex]"

--

In this formula  the letter "m" denotes the slope; the letter "c" denotes the y intercept.

(Remember, variables (letters) represent numbers! ^__^)

--

[tex]\bullet[/tex] Above is all the information we need to come up with a solution to the given problem! ^^

--

Now it's time we considered this.

"What is the slope & y intercept of the equation y=1.5x-2?"

--

If in the formula y=mx+c, the slope is m, then what is the slope in the equation y=1.5x-2?

The slope is [tex]\large\boxed{\mid \boldsymbol{1.5} \mid}}[/tex].

The y intercept is [tex]\large\boxed{\mid \boldsymbol{-2} \mid}}[/tex]

[tex]\Large\text{Great job!! c:}[/tex]

--

Hope that this helped! Best wishes!

[tex]\textsl{Reach far. Aim high. Dream big.}[/tex]

--

[tex]\underline{\underline{\underline{\overbrace{\boldsymbol{Learn\;More!!}}}}}[/tex]

✨ brainly.com/question/3837216?referrer=searchResults



Calculate the sum of each expression. Arrange the expressions in the order of their value from largest to smallest.
92 + ( -5 )
2+2

Answers

Answer:

6

Step-by-step explanation:

Answer:

92 + ( -5 ) , 2+2

Step-by-step explanation:

1) 92 + ( -5 )

= 92 - 5

= 87

2) 2 + 2 = 4

The pie chart below shows how the annual budget for a certain companies divided by department is a total budget is 15,000,000 what amount is budgeted for sales research and engineering combined

Answers

If the amount of budget for media and sales combined is $25,000,000. Then the total annual budget will be $125,000,000.

The complete question is attached below.

What is the percentage?

The quantity of anything is stated as though it were a fraction of a hundred.

The pie chart below shows how the annual budget for a certain company divided by department.

If the amount of budget for media and sales combined is $25,000,000.

Then the total annual budget will be

Let x be the total annual budget.

17% + 3% = 25,000,000 / x

       20% = 25000000 / x

      0.20 = 25000000 / x

            x = $125,000,000

More about the percentage link is given below.

https://brainly.com/question/8011401

#SPJ1

In the figure below not drawn to scale. DC is a tangent to the circle. DC = 6cm, AB = 5cm. Calculate BC. (3mks) ​

Answers

Answer:

4

Step-by-step explanation:

DA²= CB×CA

6²= y×(y+5)

36=y²+5y

y²+5y-36=0

x1+x2=-5

x1×x2=36

x1=4

x2=-9

x must be positive, then answer is 4

chek: 4×9=36

Other Questions
32 divided by 4x [16x1/2]-2 What is 60 percent of 92?1.535.5215.3O 55.249:03 mins pls help me with this PLEASE HELP 1-Spin the spinner 20 times, create a table to display the results. What is the experimental probability of spinning yellow?2-Perform another 20 spins, and record them. What is the new experimental probability of spinning yellow?3-What is the experimental probability of spinning red or yellow? Find the percent of the area under the density curve where xxx is more than 333. Youre listening to the radio and hear a piece of romantic music for orchestra. The piece is programmatic, but the entire work is contained in a single movement, which is in sonata form. Youre probably listening to a/an _________. how many feet?help me please thank u The current popularity of teams can be attributed to the fact that ________. A. they outperform individuals B. they represent a better way to use employee talents C. they reduce the need for coordination and supervision D. they aid in the performance of simple tasks that do not require diverse inputs E. they strengthen the worth of individual team players over the team cos^2 x = (1 - cos 2x)/2 A. True B. False f(z)=(1/3)h(z) = z+4/z-2evaluate (hf)(0) Can somebody help me with this questionIll very much appreciate it An upward force is applied to a 6.0kilogram box. This force displaces the box upward by 10.00 meters. What is the work done by the force on the box? A Store-It-All storage facility requires users to enter a four digit code, using numbers 0-9, to get through thegate. You need to get a box of winter clothes from your storage container, but you forgot the code.1. How many possible codes are there if the digits cannot be repeated?There arepossible codes.2. What is the probability that you will guess the correct code on your first try?P(correct code) =3. If the digits can be repeated, how many possible codes are there?There aredifferent codes. A coordinate plane with 2 lines. The first line is labeled y equals f(x) and passes through (0, 4) and (1, 1) The second line is labeled y - g(x) and is horizontal passing through (negative 2, 2), (0, 2), and (2, 2). The lines intersect at a point that is slightly to the right of (0.5, 2).If f(x) = 3x + 4 and g(x) = 2, solve for the value of x for which f(x) = g(x) is true. the importance of understanding personal assumptions behind a word is: just for fundo you believe in a God that is your heavenly father? that there is a heaven in the sky? that God is preparing a place for you for when the world ends? why or why not? three-quarters of a pile of bricks were used for a certain project. when two thirds of the reminder had been used, 50 bricks were left. how many bricks were there in the original pile? Which of the following questions would be answered by performing a scientific experiment? A. What type of food is the most enjoyable? B. What type of food is the most nutritious? C. What type of food looks the best? The surface area of the triangular prism is . What do inhibitory neurotransmitters cause ? The percentage of automobile consumers who are under 50 years of age decreased approximately linearly from 58.8% in 1980 to 50.5% in 1995. (A) Predict when the percentage will be 47%.(B) Predict the percentage in 2005.